Sr Examen

Otras calculadoras


arctan(n+3)-arctan(n+1)

Suma de la serie arctan(n+3)-arctan(n+1)



=

Solución

Ha introducido [src]
  oo                             
 __                              
 \ `                             
  )   (atan(n + 3) - atan(n + 1))
 /_,                             
n = 1                            
$$\sum_{n=1}^{\infty} \left(- \operatorname{atan}{\left(n + 1 \right)} + \operatorname{atan}{\left(n + 3 \right)}\right)$$
Sum(atan(n + 3) - atan(n + 1), (n, 1, oo))
Radio de convergencia de la serie de potencias
Se da una serie:
$$- \operatorname{atan}{\left(n + 1 \right)} + \operatorname{atan}{\left(n + 3 \right)}$$
Es la serie del tipo
$$a_{n} \left(c x - x_{0}\right)^{d n}$$
- serie de potencias.
El radio de convergencia de la serie de potencias puede calcularse por la fórmula:
$$R^{d} = \frac{x_{0} + \lim_{n \to \infty} \left|{\frac{a_{n}}{a_{n + 1}}}\right|}{c}$$
En nuestro caso
$$a_{n} = - \operatorname{atan}{\left(n + 1 \right)} + \operatorname{atan}{\left(n + 3 \right)}$$
y
$$x_{0} = 0$$
,
$$d = 0$$
,
$$c = 1$$
entonces
$$1 = \lim_{n \to \infty} \left|{\frac{\operatorname{atan}{\left(n + 1 \right)} - \operatorname{atan}{\left(n + 3 \right)}}{\operatorname{atan}{\left(n + 2 \right)} - \operatorname{atan}{\left(n + 4 \right)}}}\right|$$
Tomamos como el límite
hallamos
$$R^{0} = 1$$
Velocidad de la convergencia de la serie
Respuesta [src]
pi - atan(2) - atan(3)
$$- \operatorname{atan}{\left(3 \right)} - \operatorname{atan}{\left(2 \right)} + \pi$$
pi - atan(2) - atan(3)
Respuesta numérica [src]
0.785398163397448309615660845820
0.785398163397448309615660845820
Gráfico
Suma de la serie arctan(n+3)-arctan(n+1)

    Ejemplos de hallazgo de la suma de la serie